LSAT and Law School Admissions Forum

Get expert LSAT preparation and law school admissions advice from PowerScore Test Preparation.

User avatar
 Dave Killoran
PowerScore Staff
  • PowerScore Staff
  • Posts: 5862
  • Joined: Mar 25, 2011
|
#49526
Complete Question Explanation
(The complete setup for this game can be found here: lsat/viewtopic.php?t=18147)

The correct answer choice is (B)

Answer choice (A): From the fifth rule J must review M or W, and so this answer choice is incorrect.

Answer choice (B): This is the correct answer choice.

Answer choice (C): From the fifth rule J must review M or W, and so this answer choice is incorrect.

Answer choice (D): The third rule stipulates that L reviews the same movie as exactly one other reviewer, and so this answer choice is incorrect.

Answer choice (E): From the fifth rule J must review M or W, and so this answer choice is incorrect. In addition, G must review M.
 1800-HELPME
  • Posts: 16
  • Joined: May 19, 2017
|
#35126
I used the reviewers as my base, so question 12 was hard to figure out. Should I have used the movies as my base instead? I understand how the answer was B, but what's the most efficient way of figuring that out?

Thanks in advance :-D
Last edited by 1800-HELPME on Fri Jun 16, 2017 10:03 pm, edited 1 time in total.
User avatar
 Dave Killoran
PowerScore Staff
  • PowerScore Staff
  • Posts: 5862
  • Joined: Mar 25, 2011
|
#35216
Hi Helpme,

Thanks for the question! You've asked about one of my all-time favorite games, and it's one that really tells us a lot about selecting a base for the game, numerical distributions, and how Templates work.

As far as a base, yes, we prefer the movies as the base, primarily because it allows the numbers in the game to add extra value to the setup:

  • The game scenario and the first rule establish a numerical situation where 6 reviewers review 4 movies, with each reviewer reviewing a movie, and each movie reviewed by at least one reviewer. This “6 into 4” scenario initially has two possible distributions:

    ..... ..... ..... 3 ..... 1 ..... 1 ..... 1
    ..... ..... and
    ..... ..... ..... 2 ..... 2 ..... 1 ..... 1

    However, the second and third rules establish two blocks of two reviewers, and since neither distribution allows for four reviewers to review a movie, we can deduce that the two blocks must be separate. Hence, the 3-1-1-1 distribution is impossible, and the game must always have a 2-2-1-1 distribution.
Thus, you always know that every solution will fit into an unfixed 2-2-1-1 arrangement, which makes the setup easier to work with in our opinion.

With #12, this is a List question, and so just apply List question Technique: apply one rule at a time to each of the contending answer choices, eliminating any that violate a rule. When you do this, you find that the fifth rule eliminates two answers, and that the third and fourth rules eliminate the remaining answers:

  • Answer choice (A): From the fifth rule J must review M or W, and so this answer choice is incorrect.

    Answer choice (C): From the fifth rule J must review M or W, and so this answer choice is incorrect.

    Answer choice (D): The third rule stipulates that L reviews the same movie as exactly one other reviewer, and so this answer choice is incorrect.

    Answer choice (E): From the fourth rule G must review M, and so this answer choice is incorrect.

Thus, with four answer choices eliminated, answer choice (B) is proven correct by process of elimination.
 biskam
  • Posts: 124
  • Joined: Aug 18, 2017
|
#40493
I used the reviewers as the base instead and it took me a bit longer to find the J/W inference...

How "fatal" is it that I made this choice instead? Is there a way that other people intuitively knew that the movies as the base was the better choice based off reading the rules?

Thanks!
 Claire Horan
PowerScore Staff
  • PowerScore Staff
  • Posts: 408
  • Joined: Apr 18, 2016
|
#42215
You want to choose the base that allows for analysis of the numerical distribution and the blocks. So, for this game, the movies are the right base because from the fact of 6 people reviewing 4 movies, you know that some movies will be reviewed by multiple people. In contrast, if the base is the reviewers, each will have only one of the movies in its slot and it will be more difficult to make use of the rules. Expressed another way, my advice is to choose the variable that is most unique (repeated fewest times) to fill slots. The reviewers are the most unique because you will use each reviewer only once.

It sounds like you've hit on why it's a problem to choose the wrong base, and I don't know there are degrees of fatality involved. If you start off with the wrong base, it likely won't take you very long to realize it would have been easier the other way, so I recommend just realizing as quickly as you can and switching. Stubbornly working with a less convenient base would be a poor decision, and I can say that speaking from personal experience. :ras:

Thanks for the question!

Get the most out of your LSAT Prep Plus subscription.

Analyze and track your performance with our Testing and Analytics Package.